Important Announcement
PubHTML5 Scheduled Server Maintenance on (GMT) Sunday, June 26th, 2:00 am - 8:00 am.
PubHTML5 site will be inoperative during the times indicated!

Home Explore AFP -Workbook

AFP -Workbook

Published by International College of Financial Planning, 2020-04-20 01:54:07

Description: AFP

Search

Read the Text Version

5) You compute the value of additional life cover for Roger by considering 80% of the current household expenses, (inflation adjusted) up to Angela’s age of 55 years and further 80% of then expenses inflation-linked for the remaining period of her expected life by considering investment in debt MF schemes. This cover required to be taken as term insurance exclusive of the sum assured under current insurance policies comes to ______. [3 marks] A) ₹ 120 lakh B) ₹ 135 lakh C) ₹ 220 lakh D) ₹ 105 lakh 6) Roger’s ideal life cover has to be estimated which in case of any exigency will first repay the outstanding loans and the remaining would be invested along with the couple’s existing financial assets. Such combined corpus would be invested in a 7.5% p.a. return instrument to sustain the family’s living expenses and the specific financial goals of higher education of their children. The living expenses need to be taken as inflation-adjusted to the extent of 80% of their present household expenses for 50 yea₹ What should be this ideal cover? [4 marks] A) ₹ 147 lakh B) ₹ 165 lakh C) ₹ 180 lakh D) ₹ 230 lakh 7) Roger and Angela wish their retirement corpus, as per proposed goal, to also have a provision of gifting ₹ 50 lakh to each of their children and an additional ₹ 25 lakh towards charity to an Old Age Home at Roger’s age of 70 yea₹ The sums are at absolute values then. They also wish to provide in the corpus an additional ₹ 10,000 per month (current costs) towards healthcare after Roger’s age of 70 yea₹ You estimate the required corpus, considering the same shall be invested in investment yielding 6.5% p.a., to be _________. [3 marks] A) ₹ 3.53 crore B) ₹ 3.20 crore C) ₹ 3.78 crore D) ₹ 3.67 crore 8) You sensitize on the post-retirement parameters considered as: investment return 6.5% p.a., inflation 5% p.a., and the specified longevity as you work out retirement corpus. You stress test the same as: investment return 6% p.a., inflation 5.5% p.a., increased longevity of Roger by 5 years and of Angela by 2 years, and no further curtailment after Roger’s death. You work out the revised corpus. What additional funds need to be accumulated by Roger’s retirement age? Alternately, by what percentage the retirement expenses should be curtailed to retain this cushion? [5 marks] A) ₹ 57 lakh; 44% curtailment B) ₹ 14 lakh; 33% curtailment C) ₹ 129 lakh; 55% curtailment 97

D) ₹ 26 lakh; 36% curtailment 09) You inform Roger and Angela about the recent vehicle of taking exposure to Gold, which is Sovereign Gold Bonds (SGB). Which of the attributes about the SGB is CORRECT? [2 marks] A) The Capital Gains on redeeming these bonds on maturity are exempt from income tax B) They work like zero-coupon bonds C) The quoted prices of SGBs currently are very close to ruling Gold prices D) The redemption price on maturity is guaranteed not to be below the issue price of the respective SGB 10) Towards the marriage goal of the children, you suggest Roger to make maximum permissible subscriptions to his PPF account towards the end of every financial year and extend the account twice beyond initial maturity for terms of 5 years each with similar subscriptions. The third term of 5 years is continued without further contribution. Roger shall withdraw about 50% of accumulation for the marriage expenses of mark and the remaining for the marriage expenses of Stephanie. What are the expected individual withdrawals and shortfalls in meeting the marriage expenses? [4 marks] A) Mark ₹ 51.5 lakh, 16% shortfall; Stephanie ₹ 64.8 lakh, 18% shortfall B) Mark ₹ 47.7 lakh, 22% shortfall; Stephanie ₹ 59.7 lakh, 24.5% shortfall C) Mark ₹ 52.3 lakh, 14% shortfall; Stephanie ₹ 65.9 lakh, 17% shortfall D) Mark ₹ 45 lakh, 26% shortfall; Stephanie ₹ 56.7 lakh, 28% shortfall 11) Roger and Angela will set aside immediately a sum of ₹ 10 lakh towards setting up a fund for vacation. They will start contributing annual investments beginning April 2018 till Roger’s age of 55. The annual investment will be doubled after 14 such investments. You devise an asset allocation for the vacation fund to yield 11% p.a. for the first 15 years and 9.5% p.a. thereafter. What should be the amount of initial annual investment? [5 marks] A) ₹ 1,75,500 B) ₹ 1,23,600 C) ₹ 97,900 D) ₹ 1,15,300 12) For the higher education expenses for Mark and Stephanie, Roger starts accumulating funds with monthly investment of ₹ 20,000 in an aggressive asset allocation yielding 12% p.a. After 7 years the allocation is moderated to yield 10% p.a. and while the investment is raised to ₹ 40,000 p.m. After 12 years, the funds accumulated are shifted to suitable debt instruments from which distribution towards higher education is made as proposed. What excess/shortfall of funds you expect after 12 years by following this investment strategy? [5 marks] A) Shortfall ₹ 28.16 lakh B) Shortfall ₹ 10.12 lakh C) Excess ₹ 7.60 lakh D) Excess ₹ 12.48 lakh 98

13) Roger asks for your guidance regarding different modes of tax efficient estate planning which can help in creating and distributing family assets. You opine that a Trust would be a more appropriate option because______. [2 marks] A) there is no taxation applicable on trust income B) they have fixed rate of tax which is far lower than tax rates for individual assessees C) future capital gains tax on assets transferred to trust could be lower D) all future earnings from assets transferred to trust are exempt 14) Roger invested ₹ 4 lakh on 20th September 2016 in an Equity Mutual Fund scheme at NAV of ₹ 28.273 per unit. The scheme declared dividend of ₹ 5 per unit, the Record Date being 4th December 2016. The prevailing NAV of the scheme is ₹ 22.367 per unit. If he sells all the units of the scheme today, what would be the implication of this transaction in his IT return of AY 2018-19? [3 marks] A) ₹ 12,818 short term capital loss to be set off against capital gains in AY 2018-19 or carried in 8 subsequent years B) ₹ 83,557 short term capital loss to be set off against capital gains in AY 2018-19 only C) ₹ 83,557 short term capital loss to be set off against capital gains in AY 2018-19 or carried in 8 subsequent years D) ₹ 12,818 short term capital loss to be set off against capital gains in AY 2018-19 only 15) Roger wants to invest the maturity proceeds of all his fixed deposit investments in the 2.50 %-SGB (Sovereign Gold Bonds) of a series quoted at ₹ 2,660 per bond, at a discount of 7.3% to their issue price (issue date: 18-July-2016). Roger wishes to hold SGBs till maturity. Calculate the impact of taxation in the AY2018-19 in respect of these transactions if the SGBs could be bought at the quoted rate when FDs mature. Also evaluate capital gains on maturity in the 8-year tenure of these SGBs, if on maturity Gold price in its purest form is expected at ₹ 4,000 per gram. (CII expected in the year of maturity may be considered as 348). [5 marks] A) ₹ 1,31,556 \"Income from Other Sources\" in AY2018-19; Capital gains of 1,14,030 on maturity B) ₹ 1,31,556 \"Income from Other Sources\" in AY2018-19; Capital gains on maturity shall be tax- exempt C) ₹ 24,745 \"Income from Other Sources\" in AY2018-19; Capital gains on maturity shall be tax- exempt D) ₹ 29,477 \"Income from Other Sources\" in AY2018-19; Capital gains of 1,37,853 on maturity 99

Solutions Q1 B) professional requirement under Practice Guidelines of FPSB India Q2 D) Identify other issues that may potentially impact Roger’s ability to achieve financial goals Q3 C) ₹ 54 lakh (approx.) (Solution given below) Current value of the desired house ₹10,000,000 Expected value of new house after 5 years considering ₹13,700,867 6.5% appreciation ₹ 10000000*(1+6.5%)^5 Existing market value of the occupied house 7,500,000 years Expected market value in five years considering ₹10,275,650 p.a. 6.5% appreciation 7500000*(1+6.5%)^5 Loan outstanding on existing home to be settled ₹ 2,400,000 Principal value of 15-year loan (availed in April 2011) EMI considering 10% p.a. interest for first three years ₹25,791 p.m. PMT(10%/12,15*12,-2400000,0,0) Loan outstanding as at end March, 2014 ₹2,158,061 PV(10%/12,(15-3)*12,-25791,0,0) The average rate on loan 1.5% above the Repo rate 8.00%p.a. of 6.5% Revised EMI (average) over the next 8 years ₹23,360 p.m. (3 years till Mar’17 + 5 years until sold) PMT (8%/12,(15-3)*12,-2158061,0,0) Loan outstanding as at end March, 2022 (five ₹956,870 years from today) PV(8%/12,(15-3-3-5)*12,-23360,0,0) Amount to be set aside for tax liability, duties ₹1,000,000 and furnishing Amount that can be utilized from sale proceeds to ₹8,318,780 buy new house (10275650-956870-1000000) Amount to be financed for new house ₹5,382,087 (13700867-8318780) Q4 A) They must take Mortgage Redemption Insurance or an equivalent term insurance to cover outstanding loans Q5 D) ₹ 105 lakh (approx.) 40,000 ₹ p.m. (Solution given below) 100 Current household expenses

Annual expenses in current terms 480,000 ₹ p.a. Inflation rate 5.00% p.a. Return on Debt MF schemes 7.50% p.a. Current age of Angela 31 years PV of 80% of current expenses required till Angela's ₹7,124,721 age of 55 years PV((1+7.5%)/(1+5%)-1,55-31,- 480000*80%,0,1) Household expenses (80% of current) in the 1,238,438 55th year of Angela ₹ 480000*80%*(1+5%)^(55-31) PV at Angela's age of 55, of 80% of then living expenses for remaining 25 years ₹18,945,605 PV((1+7.5%)/(1+5%)-1,80-55,- PV of post-55 years expenses today 1238438*80%,0,1) (income stream drawn from debt funds) ₹3,339,684 18945605/(1+7.5%)^(55-31) Life cover required to the extent of covering living 10,464,405 expenses as proposed ₹ 7124721+3339684 (Approximate) ₹ 105 lakh Q6 A) ₹ 147 lakh (Solution given below) Current expenses 480,000 p.a. Rate of return to invest claim proceeds and other assets 7.50% p.a. Inflation 5.00% p.a. Living Expenses 11,420,551 ₹ (PV):1 80% of present expenses for the next 50 years PV((1+7.5%)/(1+5%)-1,50,- 480000*80%,0,1) Higher Education Expenses 1,719,344 ₹ (PV):2 Mark: ₹ 4 lakh p.a. for 4 years required after 14 years PV((1+7.5%)/(1+8%)-1,4, - at 8% escalation 400000*(1+8%)^14,0,1)/(1+7.5%)^14 Stephanie: ₹ 4 lakh p.a. for 4 years required after 17 1,743,447 ₹ (PV):3 years at 8% escalation PV((1+7.5%)/(1+8%)-1,4, - 400000*(1+8%)^17,0,1)/(1+7.5%)^17 Loans outstanding 1,785,000 ₹ (PV):4 Housing loan 305,000 ₹ (PV):5 Car loan 101

Total corpus required to meet the living and HE 16,973,342 ₹ (PV): 1 to 5 expenses and loans (PV:1 to 5) PV1+PV2+PV3+PV4+PV5 Financial Assets: Cash in bank accounts and FDs ₹ 570,000 Equity shares and Equity MF scheme investments ₹ 1,180,000 PPF A/c balance ₹ 490,000 Total of Financial Assets ₹ 2,240,000 Therefore, Life cover needed at this stage for Roger ₹ 14,733,342 16973342-2090000 (Approximate) ₹ 147 lakh Q7 B) ₹ 3.20 crore (approx.) (Solution given below) ₹ 40,000 p.m. Monthly household expenses ₹ 1,383,022 12*40000*70%*(1+5%)^(58-29) Required Annual expenses in the first year after retirement (age 58 of Roger) Rate at which corpus is invested 6.5% p.a. Inflation 5.0% p.a. PV of expenses required from Roger's age of 58 to 70 ₹ 15,369,121 (corpus: 1) PV((1+6.5%)/(1+5%)-1,12,-1383022,0,1) PV (on retirement) of Provision of Gifts and Charity at 5,871,036 age 70 ₹ (corpus:2) 12500000/(1+6.5%)^12 Additional ₹ 10,000 p.m. (current cost) at Roger's 73,920 age of 70 ₹ p.m. 10000*(1+5%)^(70-29) Additional Annual expenses to be provided for medical 887,039 care at Roger's age of 70 ₹ 73920*12 Basic Household expenses at Roger's age of 70 2,483,708 ₹ 1383022*(1+5%)^(70-58) Total Annual expenses required at Roger's age of 70 3,370,747 ₹ 887039+2483708 Corpus (at 70 of Roger) for expenses required from age 16,385,620 70 to 75 of Roger ₹ PV((1+6.5%)/(1+5%)-1,5,- 3370747,0,1) PV of this sum (computed at Roger's age of 70) on 7,696,045 Roger's retirement (at age 58) ₹ (corpus:3) 16385620/(1+6.5%)^(70-58) 102

Expenses further curtailed to 70% for Angela (Roger 3,011,415 dies at 75, Angela survives at 77) ₹ 3370747*(1+5%)^5*70% Corpus at Roger's age of 75 (death) for next three 8,907,600 years of Angela's survival# ₹ PV((1+6.5%)/(1+5%)-1,3,- 3011415,0,1) Corpus (at 58) for expenses required at Roger's age 3,053,637 75 for Angela's survival ₹ (corpus:4) 8907600/(1+6.5%)^(75-58) Total Corpus required at age 58 of Roger 31,989,838 Q8 A) ₹ 57 lakh; 44% curtailment ₹ (corp 1 to 4) #Angela (life exp. 80) survives Roger by 3 years (Solution given below) Corpus worked out in the Initial Scenario: Initial rate at which corpus is invested 6.5% p.a. Initially assumed Inflation rate 5.0% p.a. Current house hold expenses 480,000 ₹ p.a. Household expenses budgeted for retirement after 1,383,022 29 yrs (Rogers' age 58) ₹ p.a. 480000*70%*(1+5%)^29 Age of Angela on Roger's retirement (Angela is senior 60 years by 2 years) Life expectancy of Angela 80 years Retirement corpus to last (out of which last 3 years 20 years further reduced to 70%) PV of expenses: Initial 17 years (till the survival of 21,039,890 Roger up to age 75, Angela 77) ₹ PV((1+6.5%)/(1+5%)-1,17,- 1383022,0,1) PV of expenses: Balance 3 years (Angela's living 2,250,048 expenses from age 77 to 80) ₹ PV((1+6.5%)/(1+5%)-1,3, - 1383022*70%*1.05^17,0,1)/ (1+6.5%)^17 Initially worked out corpus 23,289,939 ₹ 21039890+2250048 Stress test: lower yield, higher inflation, increased longevity Retirement corpus to last (Roger's 80 with now coincide 22 years with Angela's 82) 80-58 103

Revised Yield from investing corpus 6.00% p.a. Revised Rate of inflation 5.50% p.a. Retirement corpus required 28,965,848 ₹ PV((1+6%)/(1+5.5%)-1,22,- 1383022,0,1) Cushion built in the corpus 5,675,909 ₹ 28965848-23289939 Alternately, the reduction sought in post-retire expenses (2nd Scenario) Required expenses to be withdrawn in the 1st year 1,112,016 after retirement 1383022*(23289939/28965848) Pre-retirement expenses (at the given rate of inflation 1,975,745 up to retirement) 480000*(1+5%)^29 Curtailment in expenses 43.72% 1-(1112016/1975745) Q9 C) The Capital Gains on redeeming these bonds on maturity are exempt from income tax Q10 B)Mark ₹ 47.7 lakh, 22% shortfall; Stephanie ₹ 59.7 lakh, 24.5% shortfall (Solution given below) PPF account balance as on 31-March-2017 ₹ 490,000 Account's initial maturity (opened in Dec-2011) is 1-April-2027 Number of subscriptions from 31-Mar-2018 to 10 31-Mar-2027 Number of subscriptions from 31-Mar-2028 to 10 31-Mar-2037 (2 extensions) Rate of interest assumed throughout 7.75% p.a. Maximum subscription at the end of every financial ₹ 150,000 year (for 20 years) Accumulated balance on 31-March-2037 (Mark's age ₹ 8,857,561 24, Stephanie' age 21) FV(7.75%,20,-150000,-490000,0) The account is maintained without subscription for 5 more years Accumulated balance on 31-Mar-2038 (Mark's ₹ 9,544,022 age 25 years) 8857561*(1+7.75%) 50% of accumulated amount withdrawn for Mark's ₹ 4,772,011 marriage expenses 9544022/2 Estimated expenses (current ₹ 10 lakh, escalating ₹ 6,108,808 by 9% p.a.) for Mark 1000000*(1+9%)^21 104

Shortfall in meeting Mark's marriage expenses 21.88% 1-(4772011/6108808) Remaining amount in PPF accumulated till ₹ 5,969,710 31-Mar-2041: Stephanie's marriage (9544022/2)*(1+7.75%)^3 Estimated expenses (current ₹ 10 lakh, escalating ₹ 7,911,083 by 9% p.a.) for Stephanie 1000000*(1+9%)^24 Shortfall in meeting Stephanie's marriage expenses 24.54% 1-(5969710/7911083) Q11 D) ₹ 115,300 (Solution given below) ₹150,000 The current cost of annual vacation (1-April-2017); Roger's age 29 7.00% p.a. Cost escalation provisioned in the vacation expenses ₹ 1,000,000 Lump sum invested on 1-April-2017 in the fund (Roger's age 29) 26 years Total annual investments from 1-April-2018 to 1-April-2043 (till Roger is 55) 25 years Total withdrawals from 1-April-2022 to 1-April-2046 (till Roger is 58) 29 years Total investment peiod (1-April-2017 to 1-April-2046) 11.00% p.a. Return expected from Asset allocation in the first 15 years (Initial + 14 investments) 9.50% p.a. Return expected from Asset allocation in remaining period ₹ 210,383 Vacation Expenses enumerated Vacation expenses to be first drawn after 5 year, ₹ 1,939,223 i.e. on 1-April-2022 PV((1+11%)/(1+7%)-1,11,-210383,0,1) PV of expenses (first 11 years, 1-Apr-2022 to 1,277,426 ₹ (PV:1) 1-Apr-2032) drawn from 11% return 1939223/(1+11%)^4 PV as on 1-April-2018 ₹ 442,825 150000*(1+7%)^16 Likely vacation expenses on 1-April-2033 ₹ 5,358,498 PV((1+9.5%)/(1+7%)-1,14, -442825,0,1) PV of expenses (next 14 years, 1-Apr-2033 to 1-Apr-2046) drawn from 9.5% return 105

PV as on 1-April-2018 1,135,291 ₹ (PV:2) 5358498/((1+11%)^14*(1+9.5%)) Total PV of all vacations provisioned (Pv:1 to 2) ₹ 2,412,718 1277426+1135291 Accumulation: Initial sum invested (1-April-2017) in the needed ₹ 1,000,000 fund for vacation Accumulation of initial sum as on 1-April-2018 ₹ 1,110,000 (1000000*1.11) Remaining amount to be provisioned by way of ₹ 1,302,718 annual investments 2412718-1000000 Let us assume that initial investment installment ₹ 100 (first 14) be PV of first 14 investments of ₹ 100 from 1-Apr-2018 ₹ 774.99 to 1-Apr-2031 PV(11%,14,-100,0,1) investments in the latter part, 12 annual investment ₹ 200 (from 1-Apr-2032 to 1-Apr-2043) PV of next 12 investments of ₹ 200 from ₹ 354.83 1-Apr-2032 to 1-Apr-2043 PV(9.5%,12,-200,0,1)/(1+11%)^14 PV of all 26 Annual Investments as provisioned ₹ 1,129.81 774.99+354.83 Amount of Annual Investment equivalent to the ₹ 115,304 assumption of ₹ 100 (1302718/1129.81)*100 Q12 B) Shortfall ₹ 10.12 lakh (Solution given below) Higher Edu. expenses, in current terms, of Mark ₹ 400,000 p.a. (age 4) at his age 18, 19, 20 & 21 Higher Edu. expenses, in current terms, of Stephanie ₹ 400,000 p.a. (age 1) at her age 18, 19, 20, 21 Cost escalation for higher education expenses 8.00% p.a. Expenses drawn from a fund invested in debt 7.50% p.a. instruments at Accumulation period through monthly investments 12 years in Asset Allocation Fund Present Value of Higher Education Expenses after 12 years 106

PV of Higher Edu. Expenses of Mark at his age of ₹ 4,732,399 18 (after 14 years) in Risk Free PV((1+7.5%)/(1+8%)-1,4,- 400000*(1+8%)^14,0,1) PV of such expenses after 12 years when drawn from 4,095,099 ₹ PV:1 debt investments 4732399/(1+7.5%)^2 PV of Higher Edu. Exp. of Stephanie at her age of ₹ 5,961,460 18 (after 17 years) in Risk Free PV((1+7.5%)/(1+8%)-1,4,- 400000*(1+8%)^17,0,1) PV of such expenses after 12 years when drawn from 4,152,506 ₹ PV:2 debt instruments 5961460/(1+7.5%)^5 Total PV of Higher Edu. Exp. After 12 years in Risk 8,247,605 ₹ PV:(1+2) Free instruments 4095099+4152506 Accumulation Aggressive Asset Allocation (year 1 to 7) 7 years Return expectation (aggressive) 12.00% p.a. Monthly investment ₹ 20,000 Accumulation in 7 years ₹ 2,576,027 FV((1+12%)^(1/12)-1,12*7,-20000,0,1) Moderate Asset Allocation (year 8 to 12) 5 years Return expectation (aggressive) 10.00% p.a. Monthly investment ₹ 40,000 Accumulation in 12 years ₹ 7,235,587 FV((1+10%)^(1/12)-1,12*5,-40000,- 2576027,1) Shortfall expected after 12 years ₹ -1,012,018 7235587-8247605 Q13 C) future capital gains tax on assets transferred to trust could be lower Q14 A) ₹ 12,818 short term capital loss to be set off against capital gains in AY 2018‐19 or carried in 8 subsequent years (Solution given below) Investment amount on 20‐Sep‐2016 ₹ 400,000 Investment made at a price ₹ 28.273 Units allotted Dividend received ₹ 5 per unit (RD 4‐Dec‐2016 is within 14,147.773units (400000/28.273) 3 months of purchase date) 70,739 (14147.773*5) 107

Price prevailing today (1‐Apr‐2017) 22.367 316,443 (14147.773*22.367) Redemption proceeds (within 9 months of dividend) (83,557) 316443‐400000 Short term Loss in the transaction -12,818 (‐83557+70739) Under Section 94(7) dividend stripping applies in this case Hence, short term capital loss allowable in AY 2018‐19 Note: As per Section 94(7), dividend stripping is applicable only if: 1. Shares or MF units are bought within 3 months of dividend record date 2. Shares are sold within 3 months of dividend record date/MF units are sold within 9 months of dividend record date 3. There is short term capital loss (STCL) on such sale 4. Dividend received is less than the STCL on sale If it is applicable, the amount of dividend received is deducted from the total STCL figure for shares/MF units sold. Balance will be either set‐off against capital gains, if any, or carried forward to next assessment year. Q15 C) ₹ 24,745 \"Income from Other Sources\" in AY2018‐19; Capital gains on maturity shall be tax‐ exempt (Solution given below) As on 31‐Mar‐2017 ₹ 260,663 Amount cumulated in ₹ 2 lakh, 3‐year fixed deposit made on 200000*(1+9.75%/4)^11 1‐Jul‐2014 @9.75% p.a. Amount cumulated in ₹ 1 lakh, 2‐year fixed deposit made on ₹ 117,355 1‐Jul‐2015 @9.25% p.a. 100000*(1+9.25%/4)^7 Amount cumulated in ₹ 1 lakh, 1‐year fixed deposit made on 1‐Jul‐2016 @8.75% p.a. ₹ 106,707 Maturity proceeds to be received as on 1‐Jul‐2017 100000*(1+8.75%/4)^3 Amount cumulated in ₹ 2 lakh, 3‐year fixed deposit made on ₹ 267,016 1‐Jul‐2014 @9.75% p.a. 200000*(1+9.75%/4)^12 Amount cumulated in ₹ 1 lakh, 2‐year fixed deposit made on ₹ 120,069 1‐Jul‐2015 @9.25% p.a. 100000*(1+9.25%/4)^8 Amount cumulated in ₹ 1 lakh, 1‐year fixed deposit made on ₹ 109,041 1‐Jul‐2016 @8.75% p.a. 100000*(1+8.75%/4)^4 Total maturity proceeds of Fixed Deposits ₹ 496,126 Interest accrued and receivable in the FY2017‐18 (267016+240137+109041) ₹ 11,402 Current Quoted price of SGB (issue date: 18‐July‐2016) 496126‐(260663+117355+106707) ₹ 2,660 Number of SGBs to be bought at the quoted price 186.51 bonds(round-off 186) 108

Coupon to be received on bonds (half‐yearly on 18‐Jul'17 and 18‐Jan'18) 2.50% p.a. Discount to issue price 7.30% Face value per unit of SGB ₹ 2,869 2660/(1‐7.3%) Face value of SGBs to be purchased ₹ 533,722 2869*186 interest to be received on bonds in the FY2017‐18 ₹ 13,343 533722*2.5% Total interest due to these transactions under 'Income from Other Sources' ₹ 24,745 11402+13343 Capital Gains on maturity of the SGBs in July 2024 shall be exempt from income tax 109

Case Study Sample - 2 (Urvashi) (Source: FPSB) Ms. Urvashi, aged 34 years, is employed in a senior position in a Mumbai-based firm. She has a son Suryansh aged 14 years and a daughter Dhruvi aged 9 yea₹ She is the sole guardian of her children pursuant to her recent divorce. She is currently residing in a rented house. Suryansh has just passed 8th standard while Dhruvi is studying in 3rd standard. She has approached you, a CFPCM practitioner, for preparing a Financial Plan for her family. She has plans to retire early from service at her age of 55 yea₹ She shares the following financial information with you: Salary Income (2017-2018) Annual (₹ lakh) Basic Salary : 25.00 Employer’s contribution to NPS : 2.50 HRA : 5.00 Other allowances and reimbursements : 3.00 Regular Outgoings: Monthly (₹) Basic Household Expenses : 40,000 Services availed : 18,000 School Fees : 25,000 House Rent : 35,000 Power, Telecom & Fuel : 12,000 Car Loan EMI : 18,275 Outgoings towards investment and insurance: (₹) Equity Mutual Fund17 : 25,000(Systematic Investment Plan - SIP) Debt Mutual Fund2 : 15,000 (Systematic Investment Plan - SIP) Insurance Premium3 : 38,759 Health Insurance Premium4 : 27,631 Car Insurance Premium : 8,637 Assets: (Valued on 31st March, 2017) (₹ lakh) Equity Mutual Fund schemes : 15.45 Debt Mutual Fund schemes : 5.79 Equity Shares in Demat Account : 23.92 1 Diversified open-ended growth equity schemes; started 3 years ago with initial investment of Rs. 1 lakh; monthly SIP 2 Long-term long duration debt schemes with growth option, started 2 years ago, initial investment of Rs. 1 lakh; monthly SIP 3 Total Cover Rs. 1.5 crore across three policies of Rs. 50 lakh each, all term plans having cover up to Urvashi’s age of 50, 53 and 58 year respectively; annual premium 4 Total cover Rs. 20 lakh on two policies, one is floater Rs. 10 lakh cover, the other in Urvashi’s name; annual premium 110

Equity Linked Saving Scheme5 : 3.85 Public Provident Fund (PPF) A/c6 : 6.59 Gold & Diamond Jewellery : 10.75 Car7 : 4.50 Bank Account (Salary) : 3.82 Fixed Deposits8 : 6.00 Deposit with House Owner : 3.00 Liabilities: (Outstanding on 31st March, 2016) (₹ lakh) Car loan : 5.70 You, in consultation with Urvashi, have crystallized the following financial goals, for which the strategy is to be devised and presented to Urvashi: 1. Purchase a house in the next three years costing currently ₹ 1.25 crore; provide for own funds, transfer and stamp duty expenses to the extent of 30% of market value 2. Create a pool account and manage the same to plan for basic education of both children till their respective 18 years of age; current costs are ₹ 1.5 lakh p.a. till age 14 and ₹ 2 lakh p.a. thereafter till age 18, such expenses escalate at 10% p.a. 3. Create a corpus for higher education of both children at their respective age of 18 years; ₹ 25 lakh is the outlay in current terms for each child, such costs escalate at 8% p.a. 4. Create a combined corpus for the professional courses to be pursued by children with current outlay of ₹ 25 lakh each required at their respective age of 22 years, such costs escalating at 9% p.a., such corpus sustaining till the marriage of both the children tentatively at their respective age of 27 years; marriage costs at ₹ 20 lakh per marriage, escalating at 7% p.a. 5. Retirement Corpus for post-retirement income stream equivalent to 60% current expenses arrived at by omitting rent, EMI and school fees and considering provisions for gifting a lump sum ₹ 1 crore to her children when Urvashi attains 75 years of age; a further provision of donating ₹ 1 crore posthumously to a charitable trust on reaching age 85. 6. Create a fund in 10 years for a family world tour at an estimated ₹ 10 lakh at current costs, such costs escalating at 5% p.a. Life Parameters: Urvashi’s expected life currently estimated : 85 years Assumptions regarding pre-tax returns on various asset classes (1-3 years): 1) Equity & Equity MF Schemes/Index ETFs : 11.00% p.a. 2) Balanced MF Schemes : 9.50% p.a. 3) Bonds/Govt. Securities/Debt MF Schemes : 7.50% p.a. ___________ 5 Invested ₹ 1 lakh in each of the previous three financial years in March every year 6 Account opened on 21st December 2009 7 Purchased on 1st March 2014 by availing a loan for ₹ 10 lakh (80% loan to value, 6-year, 9.5% p.a.) 8 Six Fixed Deposits each of ₹ 1 lakh at 7.75% p.a. interest, maturing on 1st date of months from April to September 2017, all deposits created from 15th September 2015 to 20th October 2015 on weekly intervals 111

4) Liquid MF Schemes : 6.00% p.a. 5) Gold & linked instruments : 6.00% p.a. 6) Real Estate Appreciation : 6.50% p.a. 7) Bank/Post Office Term Deposits (> 1 year) : 7.25% p.a. 8) Public Provident Fund/EPFO : 7.75% p.a. Assumptions Regarding Economic Factors: : 5.00% p.a. 1) Inflation : 5.50% p.a. 2) Expected Return in Risk Free Instruments Cost Inflation Index: 2001-2002 100 2004-2005 113 2007-2008 129 2010-2011 167 2013-2014 220 2016-2017 264 2002-2003 105 2005-2006 117 2008-2009 137 2011-2012 184 2014-2015 240 2017-2018 272 2003-2004 109 2006-2007 122 2009-2010 148 2012-2013 200 2015-2016 254 112

Questions 1) Urvashi asks if you can show her the actual financial plan made of another client. Under which of the following Code of Ethics you are prohibited to reveal one client’s details to othe₹ [2 marks] A) Code of Ethics of Professionalism B) Code of Ethics of Fairness C) Code of Ethics of Confidentiality D) Code of Ethics of Integrity 2) You have just defined and discussed with Urvashi the basic terms of the financial plan construction. As per Financial Planner Practice Standards, what should be your next logical step? [2 marks] A) To inform Urvashi about the terms of the engagement B) To collect the quantitative and qualitative information of Urvashi C) To define the financial goal of Urvashi D) To apprise Urvashi of your expertise in certain areas to elicit her goals accordingly 3) Urvashi wishes to avail housing loan to the extent of 70% of the value of the desired house in the next 3 yea₹ She wants to fully repay the loan by the time she intends to retire. You consider 8.5% p.a. as the average interest rate on the housing loan to be availed. She asks you by how much EMI on the loan would exceed her current monthly outgo towards house rent. [3 marks] A) ₹ 103,653 B) ₹ 44,228 C) ₹ 56,353 D) ₹ 60,703 4) Looking at Urvashi’s various insurance policies and the coverage they provide, what is the most appropriate conclusion from the following? [2 marks] A) Urvashi needs to take cover against disability and critical illness as she is the only earner in the family; other risks are well covered. B) Urvashi has to take personal accident cover which is required as she drives her own car. C) Urvashi’s life cover falls drastically after 53 years of age, she needs additional coverage till 60 years of her age. D) Urvashi needs comprehensive householder policy considering that she is single parent, is employed and is with small children. 5) Urvashi wants to create a Trust that would receive a corpus, in case of any eventuality with Urvashi’s life, towards a ₹ 70 lakh house to accommodate both children and their living expenses currently estimated at annual ₹ 9 lakh till Dhruvi attains 27 years of age. The expenses are supposed to be drawn from debt instruments. Estimate additional insurance cover to achieve thus. [3 marks] A) ₹ 36 lakh B) ₹ 84 lakh 113

C) ₹ 180 lakh D) ₹ 5 lakh 6) Urvashi’s net contribution to family in the year 2017-18 would be after an estimated tax of ₹ 7.5 lakh and 25% of such post-tax income on own consumption. This contribution is expected to increase at 5% p.a. in her service tenure. You estimate Urvashi’s income replacement considering investment yield of 8.5% p.a. What additional life cover would be needed? [4 marks] A) ₹ 1.16 crore B) ₹ 52 lakh C) ₹ 1.90 crore D) ₹ 1.45 crore 7) Urvashi’s retirement corpus is arrived at by considering current household expenses, services availed, power, telecom and fuel at her life expectancy. The investment yield at 7.5% p.a. and average inflation at 5% p.a. is considered. On a conservative note, at investment yield of 6.5% p.a. and 5 more years of expected life, what curtailment of expenses in the first year of retirement would be needed? [3 marks] A) 22% curtailment B) 12% curtailment C) 14% curtailment D) 10% curtailment 8) Urvashi’s retirement corpus as per goal needs to be accumulated by utilizing the Demat account holding along with a separate asset allocation fund. She will invest 70:30 in Equity:Debt for 10 years in this fund by beginning immediately a monthly SIP. After 10 years, the accumulated amount in asset allocation fund and the subsequent monthly investments are rebalanced 40:60 in Equity:Debt for the next 6 yea₹ After initial 16 years, the accumulations in asset allocation fund along with Demat account holdings are redeemed and transferred to a designated retirement fund yielding 6.5% p.a. The quantum of monthly investments maintained in the initial 16 years shall be doubled in the last 5 years, that is, up to retirement. This retirement fund is used for drawing expenses post- retirement. What quantum of initial monthly investment is required? [5 marks] A) ₹ 30,500 B) ₹ 63,200 C) ₹ 29,310 D) ₹ 32,100 9) Urvashi has recently heard about Inflation Indexed Bonds (IIB). She is not convinced about the real annual yield of just 1.5% in a recently issued IIB. You explain the features of such Bonds as _____. [2 marks] A) The principal amount is protected on maturity, and is repaid inflation adjusted. The annual coupons would be 1.5% of such periodically adjusted principal amount in tune with inflation index. B) The principal amount would be repaid on maturity just like other bond issues. The annual coupons would be paid at annual inflation rate plus 1.5%. 114

C) The inflation adjusted principal would be repaid on maturity. The annual coupons however would be 1.5% of the face value of the bond. D) The principal amount would be repaid on maturity just like other bond issues. The annual coupons would be 1.5% above the cumulative percentage rise in inflation index measured annually. 10) For accumulating funds for the goal of world tour, you suggest investing the maturity proceeds of each of the bank fixed deposit on the respective maturity dates in an asset allocation fund. The accumulated amount from this fund is switched to Risk free instruments three years prior to the actual usage for the purpose. What return needs to be generated from the asset allocation fund to achieve the goal? [4 marks] A) 11% p.a. B) 7.5% p.a. C) 13.25% p.a. D) 14.6% p.a. 11) Urvashi utilizes fund in her PPF account for creating a combined corpus to meet the professional course expenses of Dhruvi and later to meet her marriage expenses. She would invest ₹ 1.5 lakh in the beginning of every financial year, starting immediately, in the PPF account and extend the account for a term of 5 years with the same discipline of investment. A lump sum equivalent to 50% of the professional course charges then is withdrawn from the PPF account after which it is extended for one more term of 5 years without further contributions. What percentage of sum required for Dhruvi’s marriage would be available on the final maturity of the account? [5 marks] A) 57% B) 28% C) 45% D) 71% 12) The cash flows required for the basic and higher education expenses of her children are managed in a pool account of existing equity and debt MF schemes. They have current year provisions. You advise to switch today suitable lump sum from Equity to Debt schemes so that Debt schemes withdrawal on yearly basis is enough to meet next 3 installments of basic education expenses of Suryansh and Dhruvi. After 4 years, you again switch from Equity to Debt schemes funds equivalent to Dhruvi’s remaining years’ basic education expenses. You utilize the remaining balance in Equity schemes to meet in one lump sum Suryansh’s higher education expenses. Consider expenses required for a year to be withdrawn at the beginning of the year. What incremental SIP in Equity MF schemes needs to be started immediately to ensure this strategy works? [5 marks] A) ₹ 12,860 per month B) ₹ 19.050 per month C) ₹ 16,270 per month D) ₹ 15,120 per month 115

13) Urvashi, in case of her life contingency, is apprehensive about managing the affairs of her children. You advise her to set up a common Minor Beneficiary Trust for Suryansh and Dhruvi. You put forth the argument in favour as: [5 marks] A) Such a Trust shall protect assets transferred and shall manage them as per guidelines issued to the trustee until either or both of her children reach/es a specified age to be defined by Urvashi B) Such a Trust shall protect and manage assets for her children only until they individually reach majority, i.e. 18 years of age C) Such a Trust shall not take further resources/assets/inheritances once the benefits have been transferred to it and the guidelines specified by Urvashi for their use D) Such a Trust shall strictly prevent early distribution of assets before both Suryansh and Dhruvi attain majority, i.e. 18 years of age 14) Urvashi has decided to sell gold jewellery worth ₹ 11 lakh in April 2017. This was acquired for ₹ 2.15 lakh in FY 2003-04. She wishes to invest the proceeds of such sale after deducting tax in 2.50%-SGB (Sovereign Gold Bonds). These SGBs quote at ₹ 2,800 per bond, at a discount of 8.5% to their issue price (issue date: 28-June-2016). How these legs of transactions will reflect in her IT Return for AY2018-19? [3 marks] A) Long term capital gains of ₹ 5,63,486 ; Income from other sources ₹ 26,852 B) Long term capital gains of ₹ 5,63,486 ; Income from other sources ₹ 30,055 C) Long term capital gains of ₹ 8,85,000 ; Income from other sources ₹ 27,500 D) Long term capital gains of ₹ 5,82,478 ; Income from other sources ₹ 28,414 15) Urvashi contributes 10% of her Basic Salary to the National Pension System (NPS) Tier 1 account. Her employer also matches this contribution as 10% of her Basic Salary. She additionally contributes every year ₹ 50,000 in NPS Tier 2 account. Consider the interest on Savings Bank Account as ₹ 20,000 and that on Fixed Deposit as ₹ 25,000 during FY 2017-18. She intends to immediately buy Sovereign Gold Bonds (SGB) units for ₹ 10 lakh in a 2.5%-SGB series (issued in June 2016, with coupons payable half-yearly in June and December). The SGBs currently quote at ₹ 2,800 per unit, at a discount of 8.5% to its issued price. Calculate her income tax liability for AY 2018-19. [5 marks] A) ₹ 6,46,520 B) ₹ 7,23,770 C) ₹ 6,85,150 D) ₹ 7,46,510 116

Solutions Q1 C) Code of Ethics of Confidentiality Q2 B) To collect the quantitative and qualitative information of Urvashi Q3 D) ₹ 60,703 (Solution given below) ₹ 12,500,000 Current value of the desired house ₹15,099,370 12500000*(1+6.5%)^3 Expected value of house after 3 years considering 7% appreciation ₹ 10,569,559 15,099,370*70% Amount of loan to be availed Tenure of loan = (Urvashi's retirement age - age 18 years when loan availed) 55-37 Rate of interest on housing loan 8.50% p.a. EMI on the housing loan ₹ 95,703 Current rental outgo PMT(8.5%/12,18*12,-10569559,0,0) EMI in excess of current house rent 35,000 ₹ p.m. 60,703 ₹ p.m. 95703-35000 Q4 A) Urvashi needs to take cover against disability and critical illness as she is the only earner in the family; other risks are well covered. Q5 B) ₹ 84 lakh (Solution given below) 9,00,000 ₹ p.a. Annual Living expenses required in current terms Inflation rate 5.00% p.a. Return on risk free instruments 7.50% p.a. Current age of Dhruvi 9 years No. of years expenses required (till 27 years of 18 years age of Dhruvi) Corpus required today towards living expense ₹ 13,362,370 provisioned PV((1+7.5%)/(1+5%)-1,18,-900000,0,1) Funds required to purchase a house ₹ 10,000,000 Total Corpus for living expenses and house ₹ 23,362,370 13362370+10000000 Current insurance cover ₹ 15,000,000 Additional insurance cover required ₹ 8,362,370 23362370-15000000 (Approximate) ₹ 84 lakh 117

Q6 D) ₹ 1.45 crore ₹ 3,300,000 (Solution given below) ₹ 750,000 Urvashi's salary gross per annum ₹ 2,550,000 Tax incidence 3300000-750000 Net income in the current year ₹ 1,912,500 2550000*(1-25%) Income contribution to the family 21 years (25% self-consumption) 8.50% p.a. Remaining work life (retiring at 55, current age 34) 5.00% p.a. Investment Yield 29,508,239 PV Expected rate of increase in salary PV((1+8.5%)/(1+5%)-1,21,-1912500,0,1) Present value of future income ₹ 15,000,000 ₹ 14,508,239 Sum Assured under current term insurance 29508239-15000000 Shortfall in insurance cover ₹ 1.45 crore (Approximate) 34 yrs Q7 A) 22% curtailment 55 yrs 85 yrs (Solution given below) 840,000 ₹ p.a. Urvashi's current Age 5.00% p.a. Urvashi's retirement Age 2,340,209 ₹ p.a. Urvashi's Life expectancy 840000*(1+5%)^(55-34) Current expenses for heads considered for retirement Inflation expected pre-retirement Expenses estimated at retirement Retirement corpus calculated in the 1st calculation: 7.50% p.a. Rate of return expected 5.00% p.a. Inflation expected post-retirement 30 yrs Period for which money would be needed in 85-55 first calculation ₹ 50,952,803 Retirement corpus PV((1+7.5%)/(1+5%)-1,30,-2340209,0,1) Retirement corpus calculated in the 2nd calculation: 6.50% p.a. Rate of return expected 5.00% p.a. Inflation expected post-retirement 118

Period for which money would be needed in 35 yrs 2nd calculation 85-55+5 Retirement corpus ₹ 65,019,662 PV((1+6.5%)/(1+5%)-1,35,-2340209,0,1) Curtailment of expenses required on retirement 22% ((1-(50952803/65019662)) Q8 C) ₹ 29,310 22% (Solution given below) Current expenses for heads considered for retirement 840,000 ₹ p.a. Inflation expected throughout 5.00% p.a. Urvashi's working years (retirement at 55, 21 yrs current age 34) Living expenses needed on retirement 1,404,125 ₹ p.a. (60% of pre-retirement) 840000*60%*(1+5%)^21 No. of years retirement income stream required 30 yrs (up to age 85) Yield of designated retirement fund post retirement 6.50% p.a. Retirement Corpus estimation: Required Corpus for living expenses at age 55 years 34,551,813 ₹ PV:1 PV((1+6.5%)/(1+5%)-1,30,-1404125,0,1) ₹ 1 crore (gifts) provisioned in the corpus needed 2,837,970 ₹ PV:2 20 years later 10000000/(1+6.5%)^20 ₹ 1 crore (charity) provisioned in the corpus needed 1,511,861 ₹ PV:3 30 years later 10000000/(1+6.5%)^30 Total Corpus needed to be accumulated 38,901,644 ₹ (PV:1+2+3) Suppose, a total monthly amount of ₹ 100 is invested in the asset allocation of equity and debt components cumulatively Accumulation to meet the Retirment Corpus: Demat account Equity Shares: Current balance 2,392,000 Accumulation in 16 years, 5 years to retirement ₹ 12,703,659 (considering 11% p.a. in first 5 yrs) 2392000*(1+11%)^16 Accumulation of this up to retirement (next 5 years) ₹ 17,405,114 in 6.5% yield fund 12703659*(1+6.5%)^5 Balance to be accumulated through Asset Allocation ₹ 21,496,530 Fund up to 50 years 38901644-17405114 First Five years: Asset Allocation Equity Returns 11.00% p.a. 119

Debt returns 7.50% p.a. Equity investment per month 70 ₹ p.m. Debt investment per month 30 ₹ p.m. Equity component accumulation in 10 years ₹ 14,870 FV((1+11%)^(1/12)-1,10*12,-70,0,1) Debt component accumulation in 5 years ₹ 5,298 FV((1+7.5%)^(1/12)-1,10*12,-30,0,1) Total accumulation in asset allocation after 5 years ₹ 20,168 14870+5298 Next six years: Rebalanced Asset Allocation Rebalanced Equity component accumulated (40%) ₹ 8,067 20168*40/100 Rebalanced Debt component accumulated (60%) ₹ 12,101 20168*60/100 Revised Equity investment per month 40 ₹ p.m. Revised Debt investment per month 60 ₹ p.m. Equity component accumulation in total 16 years ₹ 19,110 FV((1+11%)^(1/12)-1,6*12,-40,- 8067,1) Debt component accumulation in total 16 years ₹ 24,100 FV((1+7.5%)^(1/12)-1,6*12,-60,-12101,1) Total accumulation in asset allocation (in 16 yrs) ₹ 43,210 19110+24100 Last Five years: 6.5% p.a. Yield Retirement Fund Monthly investment (doubled) 200 ₹ p.m. Investment accumulated up to retirement ₹ 73,342 FV((1+6.5%)^(1/12)-1,5*12,-200,- 43210,1) Actual monthly investment equivalent to ₹ 100 ₹ 29,310 (21496530/73342)*100 Q9 A) The principal amount is protected on maturity, and is repaid inflation adjusted. The annual coupons would be 1.5% of such periodically adjusted principal amount in tune with inflation index. Q10 A) 11% p.a. (Solution given below) ₹ 1,000,000 Current cost of world tour vacation Cost escalation for such vacation 5.00% p.a. Vacation fund required when due in 10 years ₹ 1,628,895 1000000*(1+5%)^10 Expected date when vacation fund is to be utilized 1-Apr-2026 120

Date of switch from the asset allocation fund to risk free 1-Apr-2023 instruments Rate of return from risk free instruments 5.50% p.a. Required value in asset allocation fund before ₹ 1,387,189 switch to risk free (3 years prior) 1628895/(1+5.50%)^3 This is to be accumulated until 1-Apr-2023 by investing ₹ 1.05 lakh in 6 installments on 1-Apr-2016, 1-May-2016, --- --- --- , 1-Sep-2016. The return to be obtained in asset allocation fund is calculated by finding xirr 1-Apr-17 -105,000 1-May-17 -105,000 1-Jun-17 -105,000 1-Jul-17 -105,000 1-Aug-17 -105,000 1-Sep-17 -105,000 1-Apr-24 1,387,189 11% XIRR(C125:C131,B125:B131) Q11 B) 28% (Solution given below) PPF account is opened on 21-Dec-2009 Initial maturity of PPF account due on 31-Mar-2025 15 years from close of FY in which a/c opened PPF account balance as on 31-Mar-2017 ₹ 659,000 Amount to be invested on 1st April every year ₹ 150,000 (beginning 1-Apr-2017) Rate of interest (expected in the long term) on PPF 7.75% p.a. account Accumulated amount on initial maturity (31-Mar-2025) ₹ 2,901,052 FV(7.75%,8,-150000,-659000,1) Current age of Dhruvi (on 1-Apr-2016) 9 years Funds for professional course required (on 1-Apr-2029) 22 years 1st extended 5-year term (from 1-Apr-2025 to 31-Mar-2030) Maturity on 31-Mar-2030 after the 1st extended term ₹ 5,156,963 with similar investments FV(7.75%,5,-150000,-2901052,1) Current cost of professional course ₹ 2,500,000 Cost escalation for professional course expenses 9% p.a. 121

Estimated outlay for professional course when due ₹ 7,664,512 (in 13 years) 2500000*(1+9%)^13 A sum equivalent to 50% of required amount ₹ 3,832,256 withdrawn from PPF account 7664512/2 Remaining amount in PPF A/c. ₹ 1,324,707 5156963-3832256 PPF account extended for 5 more years without ₹ 1,924,005 further contribution, grows to 13424707*(1+7.75%)^5 Marriage age of Dhruvi (tentatively on 1-Apr-2034) 27 years Current cost of marriage ₹ 2,000,000 Cost escalation for marriage expenses 7% p.a. Estimated outlay for marriage when due (in 18 years) ₹ 6,759,865 2000000*(1+7%)^18 Funds available as a percentage of marriage cost then 28% (1924005/6759865)*100% Q12 A) ₹ 16,270 per month incremental SIP in Equity MF schemes (Solution given below) Escalation of ₹ 1.5 lakh and ₹ 2 lakh expenses 10% p.a. Return from Debt schemes 7.5% p.a. Return from Equity schemes 11% p.a. Suryansh's current age is 14 yea₹ After the current year expenses, the required expenses in the 3- year block would be ₹ 2 lakh p.a. for 3 years (age 15,16,17). Dhruvi's current age is 9 yea₹ After the current year expenses, the required expenses in the 3-year block would be ₹ 1.5 lakh p.a. for 3 years (age 10,11,12). PV of 3 year block expenses when they are due after ₹ 1,099,599 the current year in debt PV((1+7.5%)/(1+10%)-1,3,- 350000*(1+10%),0,1)/(1+7.5%) Debt MF schemes value today ₹ 579,000 Required money to be switched from equity ₹ 520,599 schemes today 1099599-579000 Equity MF schemes value today (before switch) ₹ 1,545,000 Equity MF schemes value today (after switch) ₹ 1,024,401 1545000-520599 SIP amount in Equity schemes 25,000 ₹ p.m. Accumulated value in equity schemes after 4 years ₹ 3,050,877 FV((1+11%)^(1/12)-1,4*12,-25000,- 1024401,1) SIP amount in Debt schemes ₹ 15,000 p.m. 122

Accumulated value of SIP in debt schemes after 4 years ₹ 837,473 FV((1+7.5%)^(1/12)-1,4*12,-15000,0,1) Funds required to be available after 4 years in Debt schemes for Dhruvi's remaining five years basic education expenses (age 13,14,15,16,17) Education expenses at age 13 of Dhruvi (after 4 years) ₹ 219,615 150000*(1+10%)^4 PV of expenses for Dhruvi's basic education at her age ₹ 1,240,980 14,15,16,17 @ ₹ 2 lakh p.a. in debt schemes PV((1+7.5%)/(1+10%)-1,4,- 200000*(1+10%)^5,0,1)/(1+7.5%) after 4 years The value of funds as assessed after 4 years for ₹ 1,460,595 Dhruvi's education 219615+1240980 Shortfall in Debt schemes after 4 years ₹ 623,122 1460595-837473 Funds remaining in equity schemes after effecting switch ₹ 2,427,755 to Debt after 4 years 3050877-623122 Funds required for Suryansh’s higher education expenses ₹ 3,401,222 after 4 years 2500000*(1+8%)^4 Shortfall in total funds available, which is needed to be ₹ 973,468 accumulated through incremental SIP in equity 3401222-2427755 Therefore, incremental SIP in equity schemes ₹ 16,270 ((1+11%)^(1/12)‐1,48,0,973468, 1) Q13 A) Such a Trust shall protect assets transferred and shall manage them as per guidelines issued to the trustee until either or both of her children reach/es a specified age to be defined by Urvashi Q14 C) ₹ Long term capital gains of ₹ 5,63,486 ; Income from other sources ₹ 26,852 (Solution given below) Purchase cost of jewelry in 2003‐04 ₹ 215,000 Sale proceeds in April 2017 ₹ 1,100,000 CII for 2003‐04 109 CII for 2017‐18 272 Indexed cost of acquisition 536,514 215000*(272/109) Long‐term capital gains 563,486 1100000‐536514 LTCG Tax @20.6% 116,078 563486*20.6% Net of tax proceeds from sale of jewelry 983,922 1100000‐116078 Current quoted price of SGB 2,800 Coupon to be received on bonds (on half‐yearly basis in Jun'17 and Dec'17) 2.50% Discount to issue price 8.50% Face value of SGB 3,060 2800/(1‐8.5%) 123

Number of SGBs to be bought 351 983922/2800 interest to be received on bonds during 2017‐18 26,852 351*3060*2.5% Taxation of these transactions in AY2018‐19 Long term capital gains of ₹ 5,62,514 ; Income from other sources ₹ 26,883 Q15 B) 7,23,770 (Solution given below) Income under the head salaries: Basic 2,500,000 HRA 500,000 Less: exempt (See Note 1) (170,000) Other allowances 300,000 Employers's Contribution towards NPS (12% of Basic Salary) 250,000 Employers's Contribution towards NPS [Up to 10% of Basic Salary exempt (2500000*10%) from tax under Sec 80CCD(2)] 3,130,000 Total Income under the head salaries (2500000+500000-170000+300000+250000-250000) Income from other sources 10,000 (20000-10000) (savings account up to ₹ 10000 exempt u/s. 80TTA) fixed deposits 25,000 Income from Sovereign Gold Bonds (See Note‐2) 27,311 357*3060*2.5% Gross total income (GTI) 3,192,311 (3130000+10000+25000+27311) Less: Deductions u/s 80CCD(1) contribution NPS Tier-1 by urvashi, maximum exemption 1,500,00 u/s 80CCD(1B) contribution by Urvashi towards NPS upto ₹ 50000 50,000 u/s. 80D (restricted to maximum limit of ₹ 25,000) 25,000 Total deductions 225,000 Net Income 2,967,311 3192311-225000 Tax on net income: - up to ₹ 2,50,000 12,500 (500000-250000)*5% ₹ 2,50,001 to ₹ 5,00,000 @ 5% 100,000 (1000000-500000)*20% ₹ 5,00,001 to ₹ 10,00,000 @ 20% 124

₹ 10,00,001 and above @ 30% 590,193 (2967311-1000000)*30% Tax payable 702,693 (12500+100000+590193) Education cess and Higher education cess (2% + 1%) 21,081 (702693*3%) Total tax payable 723,774 ( 702693+21081) Rounded off 723,770 ROUND(723774,-1) Note‐1: House Rent Allowance exempted: Least of the following ‐ 500,000 Allowance Received 170,000 Rent Paid ‐ 10% of salary (35000*12)‐(10%*2500000) 1,250,000 (2500000*50%) 50% of salary Note‐2: Amount to be invested 1,000,000 Current quoted price of SGB 2,800 Coupon to be received on bonds (on half‐yearly basis in Jun'17 and Dec'17) 2.50% Discount to issue price 8.50% Face value of SGB 3,060 2800/(1‐8.5%) Number of SGBs to be bought 357.14 983922/2800 357 interest to be received on bonds during 2017‐18 27,311 (357*3060*2.5%) 125

Additional Questions 1. Mr. X’s parents are maintaining a joint Senior Citizen Saving Scheme account in which Mr. X is the sole nominee. Mr. X wants to know the status of the account after the demise of either of his parents. Which of the following is not appropriate in this context? (2) A) The surviving parent may operate the account alone. B) The surviving parent can receive the amount deposited and close the account. C) Mr. X, being the nominee will automatically replace the deceased parent in the joint account along with the surviving parent. D) The account may be continued for the remaining term with the surviving parent as the only holder And Mr. X as the nominee. 2. Mr. X has received an offer to sell his agricultural land for ₹ 11 Lakh. He wants to know, which of the following conditions does not hold for availing the exemption u/s 54 B of Income tax Act for capital gains arising on sale or transfer of agricultural land, if applicability of this section is foreseen. (3) A) The agricultural land is sold by Mr. X in his individual capacity. B) The agricultural land has been used by Mr. X or his parents for agriculture purposes during the 2- year period immediately preceding the date of sale. C) Mr. X will purchase another agricultural land from the amount of capital gains within a period of 2 years after the date of sale of agricultural land. D) Full consideration received from sale of agricultural land will be used in purchase of another Agricultural land otherwise proportionate capital gain would be eligible for exemption. 3. An investment analyst has told Mr. X to invest in a portfolio after evaluating on the following parameters - i) The performance of portfolio adjusted by the return of risk free assets over the risk of portfolio ii) Measure of the volatility in a portfolio as compared to the entire market (index) as a whole iii) Measure of how many individual elements tend to deviate from the average iv) Measure excess return on an investment over the benchmark with same degree of risk v) The proportion of variability in a portfolio compare to benchmark vi) The analyst also used a lot of terminology which confused Mr. X. He wants to know how the vii) Terminology used fits into these evaluation paramete₹ You advise the terminology, respectively, as_______. (3) B) Beta, Sharpe Ratio, Standard Deviation, Alpha, R-Squared. C) Sharpe Ratio, Beta, Standard Deviation, Alpha, R-Squared. D) Alpha, R-Squared, Standard Deviation, Sharpe Ratio, Beta. E) R-Squared, Sharpe Ratio, Standard Deviation, Alpha, Beta. 4. You have advised Mr. X to purchase a ₹ 50 lakh Life insurance Term Plan. Mr. X wants to know whether it is necessary to mention the details of his other Life Insurance policy purchased from different insurance companies. In case he fails to mention the same in the proposal form and subsequently dies due to an accident, under which principle his claim could be questioned by the 126

Insurer, if facts of the other existing insurance policy become known to the insurance company at the time of claim settlement. (3) A) Principle of Insurable Interest B) Principle of Utmost Good Faith C) Principle of Waiver and Estoppel D) Principle of Indemnity 5. Mr. X wants to make a Will and understand its procedures; you explained that the ________ is the person responsible for offering the Will for probate. (2) A) Testator B) Executor C) Lawyer D) Beneficiary 6. You have suggested a strategy which aims to invest more when the share price falls and less when the share price rises. It is done by calculating predetermined amounts for the total value of the investment in future periods and then making an investment to match these amounts at each future period. You are indicating a technique known as _________. (3) A) Rupee Cost Averaging B) Value Averaging C) Economic Cost Averaging D) Weighted Averaging 7. Mr. X wants to purchase a Child Plan from a Life Insurance company to meet Mr. X’s educational needs. He wants to know, if he gets permanent physical disabled due to accident which would hamper his income pursuits, by what means can the policy be kept in force without payment of further premium but retaining intended benefits. You advise _________. (2) A) Payor Rider B) Dreaded Disease Rider C) Living Benefit Rider D) Survivor Purchase Option Rider 8. You have explained Mr. X that while Underwriting the Insurer may counter the effects of ____________, insurers (to the extent that laws permit) ask a range of questions and may request medical or other reports on individuals who apply to buy insurance, so that the price quoted can be varied accordingly, and any unreasonably high or unpredictable risks rejected. (2) A) Moral Hazard B) Morale Hazard C) Adverse Selection D) Uberrimaefidei 9. While preparing a Financial Plan for Mr. X you have made a forecast of his present revenues and expenditures i.e. constructed a model of how his finances might perform in the near future. You have prepared a __________. (2) A) Investment Plan B) Fund Flow statement C) Cash Flow statement 127

D) Budget 10. While explaining the basics of selecting a Mutual Fund scheme you have asked Mr. X to analyse the Mutual Fund Portfolio by five main indicators that apply, one of these is __________ which is a measure of the volatility, or systematic risk, of a security or a portfolio in comparison to the market as a whole. (2) A) R-Squared B) Alpha C) Beta D) Sharpe Ratio 11. Prior to providing any Financial Planning services, you a Financial Planning practitioner and Mr. X, as your client shall mutually define the scope of the engagement. The letter of engagement would define the scope of engagement by discussing i) Identification of the service(s) to be provided ii) Financial Planning practitioner’s compensation arrangement(s) iii) Analysis and evaluation of client’s current situation iv) Determining the clients and the Financial Planning practitioner’s responsibilities; v) Establishing the duration of the engagement; vi) Determine the strategies to achieve financial goals (3) B) i), ii), iv) and v) C) ii), iii), iv) and vi) D) i), ii), iii), iv) and v) E) i), ii), v) and vi) 12. Before beginning work on Mr. X’s Financial Plan, you have drafted a “Letter of Engagement” and Sought Mr. X’s consent on the same. Mr. X asked you about relevance of such a letter. In the context of Financial Planning Profession, you explain about the “Letter of Engagement” as a _________. A) professional requirement under Code of Ethics of FPSB India B) professional requirement under Practice Guidelines of FPSB India C) legal contract as per Contract Act 1872 D) document for his personal record 13. Mr. X Gupta wants to know according to which Act his father’s estate would be distributed in case he dies Intestate. A) Hindu Succession Act, 1956, under which people belonging to Sikh, Hindu, Buddhist, Jain religion are covered B) Hindu Succession Act, 1956, under which people belonging to Sikh, Hindu, Parsi & Jain religion are covered C) Indian Succession Act, 1925, under which people belonging to Sikh, Hindu, Buddhist, Jain & Parsi religion are covered D) Indian Succession Act, 1925, under which people belonging to Sikh, Hindu, Jain, Parsi, Christian & Jews religion are covered 128

14. Mr. X wants to know what is the best instrument to get market returns over a sufficiently long period with the least recurring cost. A) Diversified Equity Growth Mutual Fund Scheme. B) Equity Index Funds. C) Equity Shares. D) Growth Option of ULIP. 15. Mr. X saw your name with CFP Marks; he wants to know different ways in which the CFP Marks in India can be written. i) CERTIFIED FINANCIAL PLANNERCM ii) CFPCM iii) CFPCM iv) C.F.P. v) CFPCM vi) Certified Financial PlannerCM B) i) & ii) C) ii), iii), vi) D) iv), v) &vi) E) ii), v) &vi) 16. Mr. X wants to invest in ULIP, but he wants to be cautious before entering a long period of contract. As Per IRDA ULIP Guidelines, if he wants to return the policy within 15 days free look period what amount would be refunded to him? A) He shall be refunded the fund value subject to deduction of expenses towards medical examination, stamp duty and proportionate risk premium for the period of cover. B) Full Premium paid is returned back to him. C) Premium paid less commission paid to intermediary is refunded to him. D) He shall be refunded the fund value. 17. Mr. X wants to know how you will ensure that information and relevant documents given to or gathered by you are securely stored? This would be is accordance to FPSB India's Rules that relate to the Code of Ethics of _____________. A) Integrity B) Diligence C) Compliance D) Professionalism 18. Before finalizing the Financial Plan, Mr. X tells you that she wants to entrust the estate issues to a solicitor, who is a friend of Mr. X. Which of the following is your best stand? A) Estate issues being substantial in the case, you maintain that the Financial Plan cannot be an integrated one if the same is outside your purview, hence decline. B) This is permissible subject to such an arrangement finding an explicit mention in the Financial Plan for the said activity. C) This is permissible subject to the advice of the solicitor being integrated into the Financial Plan and monitored along with the Plan. D) You agree to the arrangement subject to the advice of solicitor made known to you so that you modify the Financial Plan accordingly. 129

19. Mr. X, after seeing the use of CFPCM mark with your name, wants to know which entity in India is licensed to award CFPCM Certification. You tell him the name of organization as ________. (2) A)Financial Planning Standards Board India or FPSB India B) Financial Planning Standards Board of India C) Financial Planning Standards Board or FPSB D)Financial Planning Standards Board, India 20. While preparing Financial Plan for Mr. X you have ensured that all the significant recommendations are made in writing. If any significant recommendations are given orally, then confirmations have been given in writing. You have complied with Rule that relates to the Code of Ethics of ________. (2) A) Fairness B) Diligence C) Professionalism D) Compliance 21. You have advised Mr. X to buy a Term Insurance for his life and also for the life of M₹ X, he wants to know the importance of waiver of premium rider? (3) A) It is useless as there will not be any amount to be received from the Insurance Company at the time of maturity of the policy B) It is very useful as all future premiums would be waived by the Insurance Company in case the Life Assured becomes totally and permanently disabled C) It is same as Permanent Disablement rider hence need not be mentioned separately D) It is inbuilt with all the Term Insurance plans and thus need not be mentioned separately 22. You have mentioned to Mr. X that as you increase the tenure of insurance coverage, the premium charged per year will also increase. You are referring to which type of Life Insurance coverage? (2) A) Whole Life Insurance Plan B) Endowment Plan C) Unit Linked Insurance Plan D) Term Plan 23. Mr. X is seriously considering surrendering his policy of Unit Linked Pension Plan. What would be the tax implications of the surrender value received by him? (3) A) Any amount received will be tax free B) Amount received less the total premiums paid will be taxable at 10% C) Any Amount received will be added to his income and will be taxed accordingly D)Amount received less the index value of the premiums paid will be taxable at 20% 24. Mr. X wants to know the tax deduction on reinvestment of Interest on NSC. (3) A) Accrued interest (which is deemed as reinvested) also qualifies for deduction for 6 yea₹ B) Accrued interest (which is deemed as reinvested) also qualifies for deduction for first 5 year C) Accrued interest (which is deemed as reinvested) does not qualify for deduction. D) Accrued interest (which is deemed as reinvested) also qualifies for deduction for first 3 year 130

25. What would be the correct sequence to perform six steps of Financial Planning process to prepare a Financial Plan for Dr. X? (a) Developing and Presenting the Financial plan (b) Analyzing and evaluating the client’s financial status (c) Implementing the Financial Plan (d) Monitoring the Financial Plan (e) Establishing Client-Planner Relationships (f) Gather client data and determining Goals and Expectations (3) A) 1, 3, 4, 5, 2, 6 B) 6, 2, 5, 4, 3, 1 C) 6, 5, 2, 1, 3, 4 D) 5, 6, 2, 1, 3, 4 26. In the initial stage of Financial Plan preparation, you told Dr. M₹ X and also mentioned in the Financial Plan prepared that you would charge fixed fee for the Financial Plan construction and you would also earn commission on sale of recommended financial products, if the same is accepted. Which code of ethics binds the CFPCM Practitioner to disclose conflict of interests? (3) A) Objectivity B) Fairness C) Integrity D) Professionalism 27. Mr. X wants to understand the basics to analyze the financial statement of a company, he wants to know the ratios to look for in case he wants to know the profitability: (a) Return on assets (b) Return on equity (c) Total assets turnover ratio (d) Interest coverage ratio A) (a) and (b) B) (a), (c) and (d) C) (b) and (d) D) (b) only 28. Mr. X wants to know whether his Mutual Fund portfolio's returns are due to smart investment decisions by the fund manager or a result of excess risk. As he is of the opinion that one portfolio or fund can reap higher returns than its peers, it is only a good investment if those higher returns do not come with too much additional risk. He wants his Mutual Fund to give him better risk- adjusted performance. You would suggest him to look for ________ Ratio. A) Alpha B) Sharpe C) Beta D) R-Squared 29. You have suggested Mr. X to look for few important risk measures which measure different types of risks. When comparing two or more potential investments, an investor should always compare such measures. These are ________. A) CAPM, Treynor Ratio, Beta, Standard Deviation and the Sharpe Ratio 131

B) Alpha, Beta, R-squared, Standard Deviation and the Sharpe Ratio C) Quick Ratio, Alpha, Beta, Sharpe Ratio and Retention Ratio D) Debt/Equity Ratio, Current Ratio, Sharpe Ratio, Alpha and Standard Deviation 30. Mr. X has provided his assets and liabilities statement to you during the retirement planning process. The financial statement will enable you to gain an understanding of all of the following except the__________. A) Diversification of the Mr. X's assets. B) Mr. X's net cash flow. C) Mr. X's liquidity position. D) Mr. X's debt position. 31. Which of the following would not be violation of the “Principle of Integrity” in the performance of your Professional service to Mr. X? A) Exercising reasonable and prudent judgment in dealing with Mr. X. B) Making misleading claims about the scope and areas of your competence. C) Giving the impression that you are representing the views of FPSB India. D) Engaging in conduct involving dishonesty, fraud, deceit or misrepresentation. 32. You as a CFPCM Practitioner use the CFPCM mark as a proclamation to the public that you: (a) can be trusted with the clients’ financial affairs with confidence. (b) will competently fulfill the responsibilities owed to the client. (c) are governed by a professional Code of Ethics. (d) possess exhaustive knowledge of all financial matte₹ A) (a), (b) B) (a), (b), (c) C) (a), (c), (d) D) All of the above 33. You as a CERTIFIED FINANCIAL PLANNERCM Professional are required to exercise objectivity in providing services to Mr. X, your client. This means you shall be _____________. A) Impartial B) Honest C) Competent D) Diligent 34. Recently in an unfortunate event, one of Mr. X’s brothers died in a road accident. He was a bachelor and he died intestate. Mr. X’s parents were living with his deceased brother. Apart from Mr. X there are three other siblings of the deceased. Mr. X wants to know the applicable order of priority as per Hindu Succession Act for the disposition of his deceased brother’s property. A) Both parents will get the priority over all siblings of Mr. X including Mr. X himself. B) All siblings of Mr. X will get the priority over their parents. C) Mr. X’s mother will get priority over D) All of them will have equal right over the property of the deceased. 132

35. You have advised Mr. X to purchase householders Insurance Policy to insure his house as well as contents of his house. Mr. X discusses with you about the features of Householders Insurance Policy. As per you, which of the following is not a feature of Householders Insurance policy? A) Loss or damage by the insured’s domestic staff’s direct or indirect involvement in an attempted burglary is covered. B) He can make an endorsement to the policy during the currency of the policy to record alteration related to change of Insurable interest by way of sale or mortgage. C) Insured can cancel the policy during the currency of policy and get refund of premium paid (after adjustment of administrative and other exp.) on pro-rata basis. D) The Building is insured as per the re-instatement value and the contents are insured as per the market value. 36. Mr. X wants to know, what would happen to the No Claim Bonus on his car insurance after he sells his car. A) He can enjoy the No Claim Bonus on the premium for his new car if he buys it within a specified period from the date of sale of his old car B) No Claim Bonus is lost after sale of old car C) He can enjoy only 50% benefit of the No Claim Bonus on the premium for his new car if he buys it within a specified period from the date of sale of his old car D) He can avail benefit of No Claim Bonus on premium paid for Insurance taken for other purposes from same Insurer 37. You have mentioned to Mr. X that you shall ensure all information and relevant documents given to or gathered by you are securely stored to establish at any time that it has complied with the FPSB India’s Professional Standards and be available for inspection by the FPSB India when required. Such records shall be retained for seven years from the date the document was last acted upon. This is according to the Code of Ethics of __________. A) Compliance B) Professionalism C) Diligence D) Objectivity 38. At the earliest point in the relationship, you have disclosed in writing to Mr. X that you are authorized to sell or advise on a restricted range of products, and any other limitation of their capacity to serve him. You have complied with the Code of Ethics of _________. A) Compliance B) Objectivity C) Diligence D) Competence 39. M₹ X wants partition in Mr. X’s HUF, to claim her share and Mr. X’s share out of the HUF’s assets. In principle, Mr. X wants to know whether M₹ X can legally demand partition of Mr. X’s HUF as she is also one of the members in the same. A) Yes, as Mr. X has no objection B) No C) Yes, With prior permission from IT Department only D) Yes, after the death of Mr. X 133

40. A Mutual Fund agent has told Mr. X that bigger the AUM of the fund the better it is. Which of the following statements are correct? 1) The bigger the fund’s AUM, the lower the expense ratios and in that sense it could be better. 2) The bigger the fund’s AUM, less are the chances of showing break-out returns as stock buying becomes difficult without moving the price upwards. 3) The bigger the fund’s AUM the worse-off a mid & small cap fund would be, due to its limited pool of stocks. 4) The smaller the AUM of a small cap fund the better it is due to lower expenses & higher returns. B) Only 1 is correct C) Only 1 & 2 are correct D) Only 1, 2 & 3 are correct E) Only 2, 3 & 4 are correct 41. What is the correct sequence to perform six steps of Financial Planning Process to prepare a financial plan for the client? (a) Developing and Presenting the Financial plan (b) Analyzing and evaluating the client’s financial status (c) Implementing the Financial Plan (d) Monitoring the Financial Plan (e) Gather client data and determining Goals and Expectations (f) Establishing Client – Planner Relationships A) 1, 3, 4, 5, 2, 6 B) 6, 2, 5, 4, 3, 1 C) 6, 5, 2, 1, 3, 4 D) 5, 6, 2, 1, 3, 4 42. Mr. X has received few gifts in the financial year 2008-09 and he wants to know about the taxation of the same. He received a gift of ₹ 63,000 from a friend and another gift of ₹ 24,000 from his neighbor. He wants to know, what is the total taxable amount from the above receipts on which Mr. X will have to pay tax. A) ₹ 63,000, as any amount received in excess of ₹ 50,000 is taxable. B) ₹ 13,000, as the amount received over the limit of ₹ 50,000 is taxable. C) ₹ 37,000, as the total amount in excess of the limit ₹ 50,000 is taxable. D) The whole amount of ₹ 87,000, as the aggregate value of gifts received from one person or more than one person exceeds ₹ 50,000. 43. Mr. X before approaching you has also contacted another CFPCM Practitioner for the preparation of his Financial Plan. In his first meeting with the practitioner, Mr. X asked him the sources of compensation available to the practitioner by making a Financial Plan for him other than fee. But the practitioner refused to answer this question by saying that this is out of the scope of engagement. According to FPSB India’s code of ethics, the practitioner has violated Code of Ethic of _________. A) Objectivity B) Professionalism C) Fairness D) Integrity 134

44. Mr. X’s mother wants to stay with Mr. X & M₹ X on a permanent basis. Before that, she wants to settle her estate. She has decided to give her Pune house to Mr. X. The current market value of this house is ₹ 25 lakh. Since Mr. X is permanently settled in Ahmedabad and has no intention of returning to Pune, he wants to dispose of the house at current market value. From tax planning perspective, what would be the right course of action for Mr. X for transaction relating to this house property? A) Mr. X’s mother should sell this house first and then gift the sale proceed to him. B) Mr. X’s mother should gift this house to Mr. X first and then he should sell the house. C) Mr. X’s mother should make a Will Deed in favour of Mr. X first and then he should sell the house. D) Mr. X’s mother should gift this house in the name of Mr. X and M₹ X equally and then they should sell the house. 45. You have ascertained that Mr. X needs a life insurance of at least ₹ 50 lakh on top priority. At his age, a term insurance plan for a 10-year term is available for annual premium of ₹ 10,000 and for a term of 15 years the same is available for an annual premium of ₹ 12,000. He is, however, concerned of getting ‘nil’ survival benefits in case of term insurance policies, though he can currently ill afford a high premium for endowment or ‘with profit’ type of policies. A ‘return of premium’ term policy for a term of 10 years for ₹ 20 Lakh sum assured would annually cost ₹ 18,000 for his profile. He wants to know which among the following would be the most appropriate policy for him in the current circumstances. A) He should take the term plan for 15 years, which will take care of his liabilities during this period in case he dies prematurely. B) He should take the term plan for 10 years only as the premium outflow here is the least. C) He should take the ‘return of premium’ policy which would yield ₹ 2 Lakh to provide for his liabilities when he is around 55 years of age. D) He should take 10-year term policy along with a 10-year ‘return of premium’ policy to the extent of ₹ 10 lakh to optimize on premium payment while getting survival benefits. 46. Which of the following shall you avoid while providing Financial Planning services to Mr. X and M ₹ X in line with the Ethical and Professional Conduct of CFPCM Certificant entailed by FPSB India? A) Keep the client informed of developments in the field of Financial Planning. B) Advice the client in those areas in which you have competence. C) Seek council of qualified individuals for areas in which you lack adequate competence. D) Alter existing financial strategy promptly, even without confirming to client, if the change in circumstances materially impacts the client’s financial goals. 47. Mr. X wants to make some investment in the name of her mother so that she receives a regular monthly income to meet her regular expenses. He approaches you to know which of the following asset allocations you would recommend for her who will be dependent on her investments for monthly income later on. A) Fixed Deposits: 60% Post office MIS: 30% Equities: 10% B) Fixed Deposits: 40% Post office MIS: 30% Equities: 30% C) Fixed Deposits: 20% Post office MIS: 40% Equities: 40% D) Fixed Deposits: 10% Post office MIS: 40% Equities: 50% 135

48. M₹ X had earlier approached a CERTIFIED FINANCIAL PLANNERCM who suggested investment in certain financial products rather than earning minimal returns in a Savings bank account. The Financial Planner recommended such investments as he would earn commission on sale of such products. Which Code of Ethics binds a CFPCM practitioner to disclose conflict of interests? A) Objectivity B) Professionalism C) Integrity D) Fairness 49. M₹ X’s father had suffered loss from house property and loss from business and profession for the previous year and has delayed in filing his return of income before the due date. Will her father be eligible to carry forward losses at the time of filing his belated return of income? (Assuming he does not have any corresponding income to set off of losses) A) Yes, can he carry forward both B) No, cannot carry forward both C) Can carry forward only loss from house property D) Can carry forward only loss from business and profession 50. M₹ X wants to know which types of insurance she should buy, considering the assets, liabilities and her future goals. In the order of importance, which insurance should she ideally purchase? (a) Life Insurance (b) Health Insurance (c) Disability Insurance (d) Property Insurance B) 3, 1, 2, 4 C) 1, 3, 2, 4 D) 4, 3, 1, 2 E) 3, 2, 1, 4 51. M₹ X wants to buy her residential house today at a hill station as she has received a fabulous offer for a home loan. According to you, which types of insurance she should buy to cover that risk. A) Life Insurance and disability Insurance B) Disability Insurance and Accidental Insurance C) Householder’s Policy and Home loan Protection plan D) Health Insurance and Life Insurance 52. Mr. X wants to invest in a new ULIP, but he wants to be cautious before entering a long period of contract. As Per IRDA (ULIP) Guidelines, if he wants to return the policy within 15 days free look period what amount would be refunded to him? A) He shall be refunded the fund value subject to deduction of expenses towards medical examination, stamp duty and proportionate risk premium for the period of cover. B) Full Premium paid is returned back to him. C) Premium paid less commission paid to intermediary is refunded to him. D) He shall be refunded the fund value. 53. While entering into a relationship with you, Mr. X assumed that you being a CERTIFIED FINANCIAL PLANNERCM practitioner, you are fully able to take care of the execution of all aspects of his 136

Financial Plan, i.e. Taxation, Insurance, Investments, etc. As per FPSB India Code of Ethics, what is the best proposition in this context? A) This is the right assumption which can be made about all CERTIFIED FINANCIAL PLANNERCM professionals. B) The scope and limitations of the services of the CERTIFIED FINANCIAL PLANNERCM practitioner needs to be disclosed in the beginning, specifically in writing, by the professional to the client. C) A CERTIFIED FINANCIAL PLANNERCM practitioner can never take care of all aspects of a Financial Plan. D) A CERTIFIED FINANCIAL PLANNERCM practitioner is concerned with only making a Financial Plan and not its execution. 54. Mr. X has informed you that his Post Office MIS account is maturing next month. He wants to know whether this account can be extended further and, if so, for what duration? A) Cannot be extended. B) Can be extended for 24 months. C) Can be extended for 60 months. D) Can be extended for 72 months. 55. M₹ X told you that her ex-husband had purchased a life insurance policy under the MWP Act, 1874 prior to their divorce. The beneficiaries of the policy are M₹ X and their two children. M₹ X wants to know the significance and the benefits of this policy. 1) No alterations can be made by the husband once the policy is commenced 2) The proceeds of such a policy cannot be claimed by the husband or his creditors or form part of the husband’s estate 3) Alterations can be made by the wife once the policy is effected 4) The life assured is the wife A. 1, 2, 4 B. 1, 2 C. 2 D. 1, 4 56. M₹ X wants to know the importance of waiver of premium rider? You tell her that _______. (2) A) It is useless as there will not be any amount to be received from the Insurance Company at the time of maturity of the policy B) It is very useful as all future premiums would be waived by the Insurance Company in case the Life Assured becomes totally and permanently disabled C) It is same as Permanent Disablement rider hence need not be mentioned separately D) It is inbuilt with all the Term Insurance plans and thus need not be mentioned separately 57. M₹ X had earlier received calls from certain institutions offering free services of Financial Planning through their Financial Planners by subscribing to their financial products and services. She asks you the features of CERTIFIED FINANCIAL PLANNERCM practitioner that distinguishes you from other financial planne₹ You tell her that__________ (I) CFPCM practitioner has to go through an internationally accepted curriculum framework and meet the given competency profile (II) A CFPCM practitioner, once certified can follow any institution’s guidelines of Financial Planning without any recourse to FPSB India 137

(III) CFPCM practitioner has to meet stringent initial certification standards and continuing education to remain certified (IV) CFPCM practitioner has to abide by FPSB India’s Code of Ethics and professional guidelines (V) A CFPCM Certificant has to necessarily do a fee-based Financial Planning Which of the above are correct? (2) A) (II) and (V) only B) (I), (III) and (IV) only C) (I) and (II) only D) (I) and (IV) only 58. M₹ X wants to create a private trust in the name of her children. According to you, which of the following are true in case of a private trust (I) A trustee shall be any known person capable of holding property (II) A trust has to be declared by a non – testamentary instrument in writing , signed and registered or by the will of the author of the trust or of the trustee in case of an immovable property (III) A trustee would be taxed in his hands in a representative capacity where the beneficiary is a minor, lunatic or idiot or specifically entitled to receive the income from the trust (IV) The author of the trust can be the trustee himself B) (III) and (IV) C) (II) and (III) D) (II), (III) and (IV) E) (I), (II) and (IV) 59. Before beginning work on Mr. X’s Financial Plan, you have drafted a “Letter of Engagement” and sought Mr. X’s consent on the same. Mr. X asked you about relevance of such a letter. In the context of Financial Planning profession, you explain about the “Letter of Engagement” as a _________. (2) A) professional requirement under Code of Ethics of FPSB India B) professional requirement under Practice Guidelines of FPSB India C) necessary legal requirement as per Contract Act 1872 D) document for his personal record 60. Mr. X, who is a Hindu, wants to know according to which Act his father’s estate would be distributed in case he dies Intestate. (2) A) Hindu Succession Act, 1956 B) Indian Contract Act C) Indian Succession Act, 1925 D) Transfer of Property Act 61. Mr. X wants to know what the most appropriate instrument is/are to replicate exactly the equity market returns over a sufficiently long period with the least cost and risks. 1) Diversified Growth schemes of Mutual Funds 2) Equity Index Funds 3) Growth Option of ULIPs 4) Exchange Traded Funds of Equity Indices 5) A portfolio of Large Capitalized stocks 138

A) (i) and (ii) B) (ii) and (iv) C) (i) and (iii) D) (iv) and (v) 62. Mr. X saw the acronym CFPCM against your name in your business card. He wants to know about the same. You tell him that ________. A) CFP marks are owned outside the US by US based FPSB Ltd B) FPSB India is the owner of CFP marks within Indian territory C) The US based FPSB Ltd. is licensed globally to administer CFP marks D) The US based FPSB Ltd. and FPSB India are respectively licensed to issue CFP certification in US and India 63. Mr. X wants to make a Will and understand its procedures; you explained that the ________ is the person responsible for offering the Will for probate. (2) A) Testator B) Executor C) Lawyer D) Beneficiary 64. Which of the following shall you avoid while providing Financial Planning services to M₹ X in line with the Ethical and Professional Conduct required of CFPCM professionals by FPSB India? A) Alter existing financial strategy promptly in the interest of M₹ X, even without confirming to her, if the adverse developments materially impact her financial goals. B) Keep M₹ X fully informed of adverse developments affecting her financial goals and take remedial action only after her understanding of the situation and her concurrence. C) Advise M₹ X of the developments affecting her financial goals only when she comes for review of Financial Plan after the pre-determined period. D) Revise asset allocation as often as possible to ensure that the financial goals are achieved exactly as set out initially. 65. As per the practice guidelines of FPSB India followed by you, being a CERTIFIED FINANCIAL PLANNERCM practitioner, which amongst the following is the next step after defining and discussing with M₹ X the basic terms of the financial plan construction? A) To collect the general quantitative information of the prospective client B) To inform the prospective client about the terms of the engagement C) To define the financial goal of M₹ X D) To share past financial records of your existing clients with your prospective client in order to make him comfortable with number and success of your clientele funds 66. M₹ X has not bought personal accident insurance cover, though her car is covered for damages from accident. She wears seat belt and drives carefully. You, as a CFPCM professional, had your following observation about her risk management? A) M₹ X has insured the property risk; she controls some of her personal risk and retains the rest of the risk. B) M₹ X has transferred her personal risk to other drivers of the road, insured her property risk and can claim damage if accidents are caused by third party negligence. C) M₹ X has controlled her personal risk and insured her property risk. 139

D) M₹ X has not done anything to manage her risks and has to immediately go for accident and personal risk cover. She cannot rely on third party damages alone to cover the risk of the road. 67. M₹ X wants to know, which are the documents her claimant (nominee/legal heir) must send to insurance company to claim the policy benefits in case of a life insurance policy? 1) An intimation of the death of the life assured to the insurance company 2) Death certificate from local authorities 3) Completed claim forms and other forms as required by the company 4) Valid Policy of Life assurance 5) Authentic Identification that the person claiming is entitled to receive the payment B) (i), (ii), (iii), (iv) & (v) C) (ii), (iii) & (iv) D) (i), (iii) & (iv) E) (ii), (iii), (iv) & (v) 68. M₹ X wants to know if she were to meet with an accident and get permanent disability in the third year of her Term Insurance policy, what amount of the premium due in the fourth year would be payable by her. The premium being paid towards the policy is ₹ 15,000 with sum assured of ₹ 50 Lakh. A) ₹ 15,000 B) ₹ 12,000 C) ₹ 7,500 D) Nil 69. You have advised M₹ X to do Estate Planning. According to you what should be the most preferred way for her Estate Planning? A) She should prepare a Will naming her children as the sole beneficiaries as well as designate one or more guardians with their prior consent. B) She should devolve all of her personal properties to her personal HUF. C) She should prepare a Will naming her children as the sole beneficiaries in the same. D) She should transfer all of her existing properties in the names of her children and nominate her both children equally in all her legal documents. 70. In the initial stage of Financial Plan preparation, you told Mr. X and also mentioned in the Engagement Letter that you would charge fixed fee for the Financial Plan construction and you would also earn commission on sale of recommended financial products, if the same is accepted. Which code of ethics binds the CFPCM Practitioner to disclose conflict of interests? A) Objectivity B) Fairness C) Integrity D) Professionalism 71. Mr. X wants to buy a life insurance policy on the life of his father as well as both his brothe₹ However, in case of any eventuality he wants to reserve all legal rights of receiving the policy benefits in his name. He wants to know whether it is legally possible for him. A) Yes, he can buy the policy in the desired way. B) Yes, but he cannot reserve the right to receive the policy benefits. 140

C) No, in the absence of insurable interest he cannot buy life insurance policy in their name. D) He can buy the policy only in the name of his father in the desired way 72. Mr. X’s father has made a Will deed for distribution of his assets. Mr. X discusses with you regarding Probate process, as per you which is not a feature of Probate process? A) The assets are gathered, applied to pay debts, taxes and expenses of administration and distribute to those designated as beneficiaries in the Will. B) Executor or Personal Representative named in the Will is in charge of this process. C) All legal heirs will receive notices from the court to file objections. D) The court will give orders to distribute the assets to the heirs as per intestate succession Act. 73. You have disclosed in writing to Mr. X on your ability to advise and sell on a restricted range of products, and some other limitation of their capacity to serve him. You have complied with the Code of Ethics of _____________. A) Integrity B) Objectivity C) Fairness D) Diligence 74. You have already mentioned to Mr. X that you shall confirm in writing to him where a subsequent instruction given by him significantly alters the financial strategy or balance of an existing portfolio under your supervision. You have complied with the Code of Ethics of __________. A) Diligence B) Compliance C) Confidentiality D) Objectivity 75. Mr. X wants to know, what are the factors that are included in the calculation of life insurance premium rate? (a) Rate of Mortality (b) Investment earnings (c) Expenses (d) Economic condition of the country (e) Political stability A) 1, 2, 3, and 4 B) 1, 2 and 3 C) 1 and 2 D) 1, 2, 3, 4 and 5 76. Which of the following are the accepted ways globally for professionals certified by the Affiliates of FPSB Ltd. worldwide to write the certification mark against their name? (a) Certified Financial Planner (b) CFP (c) C.F.P. (d) CERTIFIED FINANCIAL PLANNER (e) C F P 141

A) 1, 2 and 3 B) 1 and 5 C) 2 and 4 D) 1 and 3 77. Which of the following usages of the certification mark owned (outside the U.S.) by FPSB Ltd. Are correct? (a) CFP Qualification (b) CFP Certification (c) CFP Education (d) CFP Professional (e) CFP Practitioner 78. Mr. X wants to know that in case he dies intestate, who among his following relatives would not get preference while his property is being devolved. A) daughter B) dad C) wife D) mother 79. You have selected a Mutual Fund scheme which has stocks in its portfolio which move together and have a high correlation. A) The Mutual Fund portfolio will have a return that is lower than the stocks included in it, but have a risk that is higher than the risk of the stocks. B) The Mutual Fund portfolio will have a return that is the average of the stocks included in it, but have a risk that is lower than the risk of the stocks. C) The Mutual Fund portfolio will have a return that is the average of the stocks included in it, but have a risk that is higher than the risk of the stocks. D) The Mutual Fund portfolio will have a return and risk, which lies in the range of risk and return of the stocks included in it. 80. The estimated value of a real estate asset in a financial statement of Mr. X, prepared by you would be based upon the: (2) A) Basis of the asset, after taking into account all straight line and accelerated depreciation. B) Mr. X's estimate of current value. C) Current replacement value of the asset. D) Value that a well informed buyer is willing to accept from a well informed seller where neither is compelled to buy or sell 81. Mr. X has asked you to give him a written assurance that if you prepare a Financial Plan for him, then in no case you would reveal any of his information to any other person, including his family membe₹ As per FPSB Code of Ethics, is it possible for you? (3) A) Yes B) No C) Yes, but with prior consent of all relevant family membe₹ D) No, because client has no authority to demand such type of assurance. 82. After working on the restructuring of the existing portfolio of Mr. X, you have recommended for a 142

major shift into equities and he has acted upon your advice implicitly. Unfortunately in the current year, equities performed badly and Mr. X’s portfolio lost almost 50% of the original investment. If he blames you for the same, then on what ground you may seek relief? (2) A) Volenti non fit injuria (to a willing person one cannot do injustice) B) Caveat emptor (let the buyer beware) C) Cuiusvis hominis est errare (every human can make a mistake) D) Ignorantia legis non excusat (ignorance of the law is no excuse) 83. Mr. X wants to know what maximum amount can be claimed with respect to each covered family member in the proposed health insurance policy (Family Floater) of Virendra’s family. (2) A) Each family member can claim upto one third amount of total sum assured. B) Both the parents can claim upto 40% each of the sum assured while the child can claim 20% of the sum assured. C) No limit is defined for individual family member subject to overall sum assured. D) None of the above 84. Mr. X is seriously concerned with the ongoing rising inflation. Taking a bitter experience of his earlier equity investments, he is keen to do some investments in debt instruments. Keeping in view the constantly rising inflation rate into account, which type of investment, from the given options, is advisable for Mr. X in the current scenario? (2) A) Bank FDR B) Long Term Bonds C) Short Term Bonds D) Floating Rate Bonds 85. Mr. X has not done any Estate Planning as of now. Even his father has not prepared any Estate Planning documents. As Mr. X is the only son of his parents, along with his 3 sisters, what is most suitable for him? (2) A) Mr. X's father should first prepare his Will and on the basis of that Will Mr. X should prepare his own Will. B) Mr. X should create his own Will without waiting for his father’s Will. C) There is no need for any Estate Planning as the family is a joint family & Mr. X is the only son of his parents. D) Mr. X should create his Will by including his father’s property but with an inbuilt provision for his sisters on account of that property. 86. Mr. X informed you that prior to consultations with you, he had contacted another CFPCM practitioner who demanded a flat remuneration of 35% of the “Assets under Management” from Mr. X for providing his services. Is there any violation of “Code of Ethics” as stipulated by FPSB India by the earlier Practioner? A) This is a matter of mutual consent between the practitioner and the client only. B) This is a violation of Code of Ethics of Professionalism. C) This is a violation of Code of Ethics of Fairness. D) This is a violation of Code of Ethics of Compliance. 87. Mr. X’s parents are senior citizens. They have no other source of income other than what they get from Mr. X per month. Mr. X wants to ensure a separate source of cash inflow for them thereby ending their dependency upon him. For this purpose he wants to deposit ₹ 5 lakh each in the name of both of his parents in Senior Citizen Saving Scheme, 2004. However before doing so he wants to know from you whether the same is allowed. (2) 143

A) No, any deposit in the said scheme should be made only from the retirement benefits of the concerned depositor. B) Yes, after the age of 60 years of depositor, source of deposit is immaterial. C) No, any deposit in the said scheme should be sourced from self-funds only. D) Yes, any person not having any source of income can make a deposit in the said scheme. 88. Mr. X is seeking your advice regarding suitability of a health insurance plan for his family. Taking into account the health status of the family, what would be your advice? (3) A) The family has good liquidity to take care of any sudden medical expenses, hence no health insurance policy is required. B) Given fairly good medical history, they should postpone taking health insurance for 5 more yea₹ C) A floater policy which covers the medical expenses of any member of the family, as well as disability insurance of Mr. X, at least, must be taken. D) Mr. X can save upto ₹ 15,000 under section 80 D by taking a suitable health cover to that extent. 89. You have suggested an investment strategy which aims to invest more when the share price or NAV falls and less when the share price or NAV rises. It is done by achieving the total targeted value of the investment by making appropriated amounts at each predetermined interval. You are indicating a technique known as _________. A) Value Averaging B) Rupee Cost Averaging C) Economic Cost Averaging D) Weighted Averaging 90. Which of the following shall you avoid while providing Financial Planning services to M₹ X in line with the Ethical and Professional Conduct required of CFPCM professionals by FPSB India? A) Alter existing financial strategy promptly in the interest of M₹ X, even without confirming to her, if the adverse developments materially impact her financial goals. B) Keep M₹ X fully informed of adverse developments affecting her financial goals and take remedial action only after her understanding of the situation and her concurrence. C) Advise M₹ X of the developments affecting her financial goals only when she comes for review of Financial Plan after the pre-determined period. D) Revise asset allocation as often as possible to ensure that the financial goals are achieved exactly as set out initially. 91. As per the practice guidelines of FPSB India followed by you, being a CERTIFIED FINANCIAL PLANNERCM practitioner, which amongst the following is the next step after defining and discussing with M₹ X the basic terms of the financial plan construction? A) To collect the general quantitative information of the prospective client B) To inform the prospective client about the terms of the engagement C) To define the financial goal of M₹ X D) To share past financial records of your existing clients with your prospective client in order to make him comfortable with number and success of your clientele funds 92. M₹ X has not bought personal accident insurance cover, though her car is covered for damages from accident. She wears seat belt and drives carefully. You, as a CFPCM professional, had your following observation about her risk management? 144

A) M₹ X has insured the property risk; she controls some of her personal risk and retains the rest of the risk. B) M₹ X has transferred her personal risk to other drivers of the road, insured her property risk and can claim damage if accidents are caused by third party negligence. C) M₹ X has controlled her personal risk and insured her property risk. D) M₹ X has not done anything to manage her risks and has to immediately go for accident and personal risk cover. She cannot rely on third party damages alone to cover the risk of the road 93. M₹ X wants to know, which are the documents her claimant (nominee/legal heir) must send to insurance company to claim the policy benefits in case of a life insurance policy? 1) An intimation of the death of the life assured to the insurance company 2) Death certificate from local authorities 3) Completed claim forms and other forms as required by the company 4) Valid Policy of Life assurance 5) Authentic Identification that the person claiming is entitled to receive the payment A) (i), (ii), (iii), (iv) & (v) B) (ii), (iii) & (iv) C) (i), (iii) & (iv) D) (ii), (iii), (iv) & (v) 94. M₹ X wants to know if she were to meet with an accident and get permanent disability in the third year of her Term Insurance policy, what amount of the premium due in the fourth year would be payable by her. The premium being paid towards the policy is ₹ 15,000 with sum assured of ₹ 50 Lakh. A) ₹ 15,000 B) ₹ 12,000 C) ₹ 7,500 D) Nil 95. You have advised M₹ X to do Estate Planning. According to you what should be the most preferred way for her Estate Planning? A) She should prepare a Will naming her children as the sole beneficiaries as well as designate one or more guardians with their prior consent. B) She should devolve all of her personal properties to her personal HUF. C) She should prepare a Will naming her children as the sole beneficiaries in the same. D) She should transfer all of her existing properties in the names of her children and nominate her both children equally in all her legal documents. 96. Mr. X has told you that one of his friends was consulting a financial advisor who moved a major chunk of his friend’s investments into equities when markets were at their peak resulting in loss of a major portion of that investment. His friend is very upset with that as he was not taken into confidence before taking this step. You convince Mr. X that you, being a CFPCM Certificant, are bound by Code of Ethics of Diligence. Which of the following is not a feature of the said Code? While preparing oral and written recommendations, a member of FPSB India shall _________. A) conduct or have access to research on financial strategies and products that may be appropriate to achieve the client’s identified needs and objectives 145

B) develop a suitable financial strategy or plan for the client based on the relevant information collected and analyzed C) take reasonable steps to place the client in a position to comprehend the recommendations and the basis thereof D) build within the recommendations suitable ‘stop loss’ situations to avoid losses of high magnitude 97. Mr. X wants to buy a life insurance policy on the life of his father as well as both his brother However, in case of any eventuality he wants to reserve all legal rights of receiving the policy benefits in his name. He wants to know whether it is legally possible for him. A) Yes, he can buy the policy in the desired way. B) Yes, but he cannot reserve the right to receive the policy benefits. C) No, in the absence of insurable interest he cannot buy life insurance policy in their name. D) He can buy the policy only in the name of his father in the desired way. 98. Mr. X’s father-in-law is a well-established businessman and M₹ X is their only child. He wants to include Mr. X as a member in their HUF. Is it possible? A) Yes B) No C) Yes, but with prior permission from IT department D) Yes, but first Mr. X’s father-in-law should prepare a non-revocable Will in favour of Mr. X 99. Mr. X wants to know from you, what is the status of his nomination in financial assets of his M₹ X as she has died intestate? A) Nomination shows that Mr. X will get the full custody and the sole ownership as well of the financial assets. B) Nominee has only the right to receive the proceeds of financial assets; the distribution of such assets to legal heirs may take place later as per Succession law. C) Nominee’s role in nomination is equivalent to the role of executor applied in case of Will. D) Nomination gives right of share of ownership even to person who has no blood relation with deceased. 100. Mr. X wants to know, the advantage from tax angle by investing in an Equity Mutual Fund scheme vis-a vis direct investment in equity shares? A) Mutual Fund scheme pay tax-free dividend, while dividends paid directly by companies are taxable B) Mutual Fund schemes are not subject to short-term Capital Gains tax on sale of shares, such gains may be passed on to investors in the form of tax free dividends C) Equity Mutual Fund schemes do not pay dividend distribution tax, while dividend payments from companies are subject to distribution tax D) Mutual fund redemptions are exempt from capital gains tax 101. You have found that all the stocks in M₹ X’s portfolio move together and have a high correlation. How will that impact the risk and return of the portfolio? A) The portfolio will have a return that is the average of the stocks included in it, but have a risk that is lower than the risk of the stocks. 146


Like this book? You can publish your book online for free in a few minutes!
Create your own flipbook